Instructions Pages 1 And 2 Of This Application Should Be Completed Using This Electr: Fill & Download for Free

GET FORM

Download the form

The Guide of drawing up Instructions Pages 1 And 2 Of This Application Should Be Completed Using This Electr Online

If you take an interest in Edit and create a Instructions Pages 1 And 2 Of This Application Should Be Completed Using This Electr, heare are the steps you need to follow:

  • Hit the "Get Form" Button on this page.
  • Wait in a petient way for the upload of your Instructions Pages 1 And 2 Of This Application Should Be Completed Using This Electr.
  • You can erase, text, sign or highlight as what you want.
  • Click "Download" to save the changes.
Get Form

Download the form

A Revolutionary Tool to Edit and Create Instructions Pages 1 And 2 Of This Application Should Be Completed Using This Electr

Edit or Convert Your Instructions Pages 1 And 2 Of This Application Should Be Completed Using This Electr in Minutes

Get Form

Download the form

How to Easily Edit Instructions Pages 1 And 2 Of This Application Should Be Completed Using This Electr Online

CocoDoc has made it easier for people to Modify their important documents across online website. They can easily Edit according to their ideas. To know the process of editing PDF document or application across the online platform, you need to follow these simple steps:

  • Open the website of CocoDoc on their device's browser.
  • Hit "Edit PDF Online" button and Choose the PDF file from the device without even logging in through an account.
  • Edit your PDF for free by using this toolbar.
  • Once done, they can save the document from the platform.
  • Once the document is edited using the online platform, the user can easily export the document of your choice. CocoDoc ensures that you are provided with the best environment for fulfiling the PDF documents.

How to Edit and Download Instructions Pages 1 And 2 Of This Application Should Be Completed Using This Electr on Windows

Windows users are very common throughout the world. They have met thousands of applications that have offered them services in managing PDF documents. However, they have always missed an important feature within these applications. CocoDoc aims at provide Windows users the ultimate experience of editing their documents across their online interface.

The procedure of modifying a PDF document with CocoDoc is easy. You need to follow these steps.

  • Select and Install CocoDoc from your Windows Store.
  • Open the software to Select the PDF file from your Windows device and proceed toward editing the document.
  • Modify the PDF file with the appropriate toolkit provided at CocoDoc.
  • Over completion, Hit "Download" to conserve the changes.

A Guide of Editing Instructions Pages 1 And 2 Of This Application Should Be Completed Using This Electr on Mac

CocoDoc has brought an impressive solution for people who own a Mac. It has allowed them to have their documents edited quickly. Mac users can create fillable PDF forms with the help of the online platform provided by CocoDoc.

For understanding the process of editing document with CocoDoc, you should look across the steps presented as follows:

  • Install CocoDoc on you Mac to get started.
  • Once the tool is opened, the user can upload their PDF file from the Mac in seconds.
  • Drag and Drop the file, or choose file by mouse-clicking "Choose File" button and start editing.
  • save the file on your device.

Mac users can export their resulting files in various ways. They can either download it across their device, add it into cloud storage, and even share it with other personnel through email. They are provided with the opportunity of editting file through multiple ways without downloading any tool within their device.

A Guide of Editing Instructions Pages 1 And 2 Of This Application Should Be Completed Using This Electr on G Suite

Google Workplace is a powerful platform that has connected officials of a single workplace in a unique manner. While allowing users to share file across the platform, they are interconnected in covering all major tasks that can be carried out within a physical workplace.

follow the steps to eidt Instructions Pages 1 And 2 Of This Application Should Be Completed Using This Electr on G Suite

  • move toward Google Workspace Marketplace and Install CocoDoc add-on.
  • Upload the file and tab on "Open with" in Google Drive.
  • Moving forward to edit the document with the CocoDoc present in the PDF editing window.
  • When the file is edited at last, download and save it through the platform.

PDF Editor FAQ

Has anyone successfully applied for Global Entry while in the US on an H1B visa?

I have a Premium Credit card which has a TSA credit allowance of 100$ for every 4 years. So I decided to apply for global entry and Here’s my experience.Jan 2019:I filled in the global entry application on Trusted travelers Program website(Official Trusted Traveler Program Website) and Paid the 100$ Fee using my credit card.I have got a credit for 100$ used on TTP website automatically within a week.I have a trip to India coming up soon and I registered on the http://www.passportindia.gov.in website and signed up for Background verification for GEP program.(Follow the instructions here, How to Apply for Passport)Feb 2019:I paid 500 rupees(I think) as fees and Scheduled an appointment(slots were almost immediately available in Visakhapatnam passport seva kendram, Andhra pradesh). Carry your passport and Address proofs and you should be all set within an Hour. Then your GEP verification will give you a conditional approval with a police verification due soon. In my case the police arrived to my home address(as on passport and declaration) and verified.Here’s the List of accepted Documents:Old passport in original with self-attested photocopy of its first two and last two pages including ECR/Non-ECR page and page of observation (if any). Proof of Present Address List of Acceptable Documents: (i) Water Bill/Telephone (landline or post paid mobile bill)/Electricity bill (ii) Photo passbook of running Bank Account (Scheduled Public Sector Banks, Scheduled Private Sector Indian Banks and Regional Rural Banks only)/Income Tax Assessment Order/Election Commission Photo ID card/Proof of Gas Connection (iii) Certificate from Employer of reputed companies on letter head (Only public limited companies can give address proof on company letter head along with seal. Computerized print-outs shall not be entertained.) (iv) Parent's passport copy, in case of minors(First and last page) (v) Applicant's Aadhaar card (vi) Rent Agreement Note: 1. This document is applicable only if applicant's present address is different from that in the existing passport 2. Applicants are required to submit the proof of address of the present address only, irrespective of the date from which he/she has been residing at the given address. 3. Aadhaar letter/card or the e-Aadhaar (an electronically generated letter from the website of UIDAI), as the case may be, will be accepted as Proof of Address (POA) and Proof of Photo-Identity (POI) for availing passport related services. Acceptance of Aadhaar as PoA and PoI would be subject to successful validation with Aadhaar database. 4. Any of the remaining documents containing address Out of 12 documents listed under Tatkaal application, could also be accepted as proof of residence if such documents have the same present residential address as given by the applicant in the Passport Application Form.There was no update what-so-ever, complete silence for the next few months. Application. The GEP Application Status on the Passport Seva, Ministry of External Affairs, Government of India was as follows:GEP verification Process has been initiated on your application. Once inputs from all the concerned government authorities are received, the report will be shared with US customs and boarder protection (CBP). No queries would be entertained in relation to your GEP Application status at the passport offices and PSP call center. you may contact CBP for further updates on the processing of your GEP ApplicationJune 2019:My US Visa stamping has expired meanwhile(the Visa info i used for this application). I have an extension filed for Change of status and approved. But I haven’t gone to stamping until oct 2019.August 2019:I tried to contact PSP call center, emailed the passport seva kendra and also contacted the CBP customer service regarding the status of my application. there was no response what so ever. I assumed this is a flop process and forgot about this after.Sept 2019:Had to go on an international trip and went stamping. got approved.Oct 2019:I flew into the united states and I was going through the customs at the port of entry. After conversation and regular customs stuff with the CBP/DHS officer, The officer informed me that my Global Entry program application is now partially approved and the officer asked me to take a visit to the Nexus/GEP/Sentri Enrollment Center. I’m quiet surprised almost forgot about this.I’m assuming because my visa(Visa information used for initial application) has expired my GEP application was stuck and as soon as my new visa information was linked onto my profile by the CBP/DHS officer, I think my GEP application was conditionally approved.I scheduled an interview at Blaine Enrollment Center, WA. (slots were available with a week) and had an interview.Interview:Take all the necessary documents(usual customs stuff) and be there for appointment. The conversation with the CBP interviewer was, He asked me a few question to verify my identity and questions like any updates to your personal information since you entered your details on the application in Jan 2019?I had a change of address, reported that. Change in visa status reported that, Change in job title reported that.. etc.He asked me usual stuff like if I ever was involved in a crime? If I ever got detained at a customs boarder check? etc..the interview was quick and general conversation lasted for about 5 to 10 mins.The CBP officer gave me a Known Traveler Number. I got a Approval notice(on the ttp portal) for GEP within 48 hours from my appointment at Blaine Enrollment Center). I was also given the TSA pre benefit and this is valid for 5 years effective immediately(even though I haven’t received my Global Entry Card).I am yet to, but I was told I would receive my GEP card with a couple of weeks at my home address.In my case the whole process took about 10 months. Maybe because I had an expired visa for a few months but I think this could be quicker if you have a valid visa.Edit:5/18/20: I was told I would receive a Global Entry card via mail during my Interview. I never received such card so far(been over 6 months). I think they were wrong about the card. All you’ll have is a KTN(Known Traveler Number) and that’s it. It will be liked to your credentials (i’m assuming) they’ll be able to verify you using your identification and the KTN.Make sure you update your airline profile with your KTN. They won’t know unless you tell them and you should provide your KTN when you book your ticket to have the TSA Pre on your boarding pass. if your boarding pass doesn’t say TSA Pre, you cant use TSA PRE and there’s no way for you to get that added right before the flight.

What is your review of University of California, Riverside?

★★★★The Case for an Overlooked University2013-09-16While searching for colleges to apply to, you may have heard about the University of California, Riverside, or UCR. Often an overlooked campus for its lack of top-achievers and applicant ignorance to the point of knowing nothing besides there being a river nearby (which there isn't), UC Riverside is definitely a campus worth considering.FactsLet's start with the facts. UCR is the sixth out of the ten UCs to be founded. It hosts 18,539 undergraduate and 2,466 graduate students. It sits in the Inland Empire, fifty-six miles east of Los Angeles. It's U.S. News ranking is 112th nationally, or 55th among public schools. [1] It also ranks 10th in Leiden University's Natural Science and Engineering rankings and 2nd in the Washington Monthly National Universities rankings. [3] All rankings were taken from 2013.UC RejectsNow when hearing about UCR, you may have heard the term "UC Rejects". This is quite the insult, but it was in fact justified. The exact nature of this term refers to the "referral" pool of UC applicants--those who meet the a-g minimum requirements to apply, but are not accepted to any of the UCs they apply to. UCs low on applicants offer these referral students an application, or "referral" to their school. In the past, UCRiverside took part in the referral program, but from 2011 on, UC Riverside has enough applicants such that they do not accept referral students anymore. [4] Therefore, the term "UC Rejects" is no longer justified.AcademicsAcademics are the most important part of any school comparison. When picking a school, the most important question should be "What is the quality of the undergraduate program for my major?" And like two t-shirts at Ross, two four-year academic programs are best compared by experiencing both of them and comparing from the student perspective. Unfortunately, most people do not have eight years to compare two different programs firsthand.I am a student currently enrolled in UCR's electrical engineering program, or EE for short. I will be starting year four in the fall. Even now, I am curious as to whether my education would have been better had I gone to another school, so I did the next best thing to experiencing two programs--I experienced one and read about another.During summer 2013, I decided to do my own investigation. I went to UC San Diego's advising office and obtained a copy of their EE program description with course listing. I also went to a professor's office and looked at the final exam for an "electronic circuits" class. Comparison between the UCR and UCSD EE program descriptions did not yield much--sure there was "multivariable calculus" on both programs, but this told me nothing of the actual rigor of the courses. For the more specific final exam that I reviewed, I did see some content that was not covered at my school, such as "frequency response of differential amplifiers," but there was also content in my course that was not on the UCSD final exam. In short, if it is hard for me to compare having experienced one of the programs, it would be nearly impossible for someone who has experienced neither program to compare well.Fortunately, there exists a solution to this difficulty in program comparison. A set of standards set forth by the Accreditation Board for Engineering and Technology (ABET) sets very specific standards on the material that should be taught in every course. For example, in the "Logic Design" standards, students should be able to "understand Boolean algebra", "design simple combinational logics" and be familiar with concepts such as "SR Latch, D Flip-Flop". [7] In other words, any school that is ABET accredited will exhibit the same material in all of their courses. As of this writing, all nine undergraduate UCs except UC Merced are ABET accredited. In fact, in November 2012, UCR's ABET accreditation was renewed for the maximum length of six years, with the visitors saying that the programs were "among the best they have visited". [8]These ABET requirements are rigorous as well--they are not like SFUSD's bare minimum graduation requirements, being so lax that the UC and CSU systems had to define their own standards. No, these standards are credible enough such that one of the requirements to obtaining "Professional Engineer" certification is to "Earn a four-year degree in engineering from an accredited engineering program" (ABET) [9] And with the programs being so difficult to compare and the ABET standards so definitive, U.S. News' only requirement for being on "Best Undergraduate Engineering Programs Rankings" is "a school must have an undergraduate engineering program accredited by ABET." [10] As far as how they are ranked, "These undergrad engineering programs were ranked solely on a peer assessment survey conducted in spring 2013", peer assessment being senior faculty and deans, whatever their perspective is worth on an undergraduate education.As for outside the engineering college, the situation is very similar. From the perspective of lower-division psychology and biology courses that I have taken, the standards are the same across schools. This is manifested by the textbook schools use. At both City College of San Francisco and UCR, the "Introduction to Psychology" courses that I took both used "Psychology" by David G. Meyers, the same textbook used in AP Psychology at Lowell. Leo, a personal friend of mine, told me Meyers is also used at UC Irvine. In my "Cell and molecular biology" course, the textbook was "Campbell Biology", the same one used in Lowell's AP Biology course. These textbooks are large and contain much material, but they are nonetheless the same across schools.What this ABET and textbook standardization means is that the material taught across schools is the same with very minor differences. This is especially true in the University of California, where lectures are only three hours per week per course, leaving most of the learning to be done at home, from the textbook. What will differ across campuses is the performance of the students. If your SAT score and GPA are well below the average for that campus, that is a good indication that you will be in the lower half of the class. This can be motivating--but at the same time it can be demoralizing. The right balance is needed.FacilitiesIn addition to great academics, UCR has excellent facilities that will benefit all students--not just engineering majors. All classrooms are very modern and come equipped with a ceiling mounted projector (often three), audience response system, desktop computer and wireless microphone. The newest classrooms even come with touch screens. Also present are things I wished Lowell had when I graduated in 2010: namely, online class enrollment and teacher evaluation forms. All of this allows for excellent instruction.The school is also constantly being improved. A million-dollar track was completed in 2012 as a result of the UCR episode of television series "Undercover Boss". [5] In the case of the student recreation center, where you may want to run, lift weights or learn martial arts, UCR's recreation center offers all the same facilities as UCSD's RiMAC, but with a better layout. (I have been to both) A recreation center add-on, due for 2014 completion, will contain triple the machines, indoor pool and more. [6]For engineering majors specifically, a new materials science and engineering building was completed in 2010, with another building scheduled for the future. Engineers are also supported by the UCR Library, where they have access to online research journals such as IEEE Xplore.EmploymentAs far as UCR grads being hired, I see my peers being hired full-time from large defense companies like Boeing, Northrop Grumman and other organizations like NASA. Even more obtain internships at Aerovironment, Los Alamos National Laboratory, and other engineering firms. In other words, companies look everywhere for talent.ConclusionNow after all of this, your opinion may not change. Relatively speaking, UC Berkeley, Los Angeles, and San Diego are still more selective and more prestigious universities overall. There is nothing wrong with applying to them. However, if your GPA and SAT score do not put you in a favorable position for those schools, know that there are several places, Riverside included, where you may still obtain a high quality education.[1] University of California--Riverside[2] Page on scrible.com[3] National University Rankings 2013[4] Students Clamor to Attend UCR[5] UC Riverside Track Facility to be Dedicated on Friday, March 2[6] UCR Recreation: Home[7] Page on ucr.edu[8] Page on scrible.com[9] What is a PE?[10] Best Undergraduate Engineering Programs

Can anyone provide detailed solutions of the Prelims 2018?

UPSC 2019 GS PAPERSET- D1. With reference to Asian Infrastructure Investment Bank (AIIB), consider the following statements1. AIIB has more than 80 member nations.2. India is the largest shareholder in AIIB.3. AIIB does not have any members from outside Asia.Which of the statements given above is / are correct?(a) 1 only(b) 2 and 3 only(c) 1 and 3 only(d) 1, 2 and 3Sol. 1 (a) 1 onlySource: PrepMate International Organizations and Bilateral Relations, Chapter 4, Page 96Statement 1 is correct.Statements 2 and 3 are incorrect.2. What was the purpose of Inter-Creditor Agreement signed by Indian banks and financial institutions recently?(a) To lessen the Government of India's perennial burden of fiscal deficit and current account deficit(b) To support the infrastructure projects of Central and State Governments(c) To act as independent regulator in case of applications for loans of Rs. 50 crore or more(d) To aim at faster resolution of stressed assets of Rs. 50 crore or more which are-under consortium lendingSol. 2 (d) To aim at faster resolution of stressed assets of Rs. 50 crore or more which are-under consortium lendingSource: PrepMate Current Affairs3. The Chairman of public sector banks are selected by the(a) Banks Board Bureau(b) Reserve Bank of India(c) Union Ministry of Finance(d) Management of concerned bankSol. 3 (a) Banks Board BureauSource: PrepMate Current AffairsIt is to be noted that Banks Board Bureau recommends the appointment of a particular person as Chairman of a public sector bank. Thus, Chairman of public sector banks is selected by Banks Board Bureau. However, the final appointment is made by Union Government.4. Consider the following statements:1. Petroleum and Natural Gas Regulatory Board (PNGRB) is the first regulatory body set up by the Government of India.2. One of the tasks of PNGRB is to ensure competitive markets for gas.3. Appeals against the decisions of PNGRB go before the Appellate Tribunals for Electricity.Which of the statements given above are correct?(a) 1 and 2 only(b) 2 and 3 only(c) 1 and 3 only(d) 1, 2 and 3Sol. 4 (b) 2 and 3 onlySource: PrepMate Current affairsStatement 1 is incorrect. There are many regulatory bodies in India which are working since independence. Some examples of these bodies are Reserve Bank of India, Railway board, etc.Once we come to know that statement 1 is incorrect. We can eliminate all the answer choices which have statement 1 in them. Thus, we are left with option (b) only.5. With reference to communication technologies, what is/are the difference / differences between LTE (Long-Term Evolution) and VoLTE (Voice over Long-Term Evolution)?1. LTE is commonly marketed as 3G and VoLTE is commonly marketed as advanced 3G.2. LTE is data-only technology and VoLTE is voice-only technology.Select the correct answer using the code given below.(a) 1 only(b) 2 only(c) Both 1 and 2(d) Neither 1 nor 2Sol.5 (d) Neither 1 nor 2Source: PrepMate Science & Technology, Chapter 13 Communication Technology, Page 190Both the Statements are incorrect.6. Which of the following statements is / are correct regarding the Maternity Benefit (Amendment) Act, 2017?1. Pregnant women are entitled for three months pre-delivery and three months post-delivery paid leave.2. Enterprises with creches must allow the mother minimum six creche visits daily.3. Women with two children get reduced entitlements.Select the correct answer using the code given below.(a) 1 and 2 only(b) 2 only(c) 3 only(d) 1, 2 and 3Sol.6 (c) 3 onlySource: PrepMate Current AffairsStatements 1 and 2 are incorrect. Only Statement 3 is correct.7. Which one of the following is not a sub-index of the World Bank's 'Ease of Doing Business Index'?(a) Maintenance of law and order(b) Paying taxes(c) Registering property(d) Dealing with construction permitsSol. 7 (a) Maintenance of law and orderSource: PrepMate International Organizations and Bilateral Relations, Chapter 7, Page 1678. In India, 'extended producer responsibility' was introduced as an important feature in which of the following?(a) The Bio-medical Waste (Management and Handling) Rules, 1998(b) The Recycled plastic (Manufacturing and Usage) Rules, 1999(c) The e-Waste (Management and Handling) Rules, 2011(d) The Food Safety and Standard Regulations, 2011Sol. 8 (c) The e-Waste (Management and Handling) Rules, 2011Source: PrepMate Environment & Biodiversity, Chapter 5, Page 909. The economic cost of food grains to the Food Corporation of India is Minimum Support Price and bonus (if any) paid to the farmers plus(a) transportation cost only(b) interest cost only(c) procurement incidentals and distribution cost(d) procurement incidentals and charges for godownsSol. 9 (c) procurement incidentals and distribution costSource: PrepMate Economics, Chapter 7, Page 12610. In the context of any country, which one of the following would be considered as part of its social capital?(a) The proportion of literates in the population(b) The stock of its buildings, other infrastructure and machines(c) The size of population in the working age group(d) The level of mutual trust and harmony in the societySol. 10 (d) The level of mutual trust and harmony in the societyTopic: MiscellaneousThis question is unexpected. However, UPSC does have tendency to ask such questions. Social capital broadly refers to those factors which effect functioning of social groups. Such factors include interpersonal relationships, a shared sense of identity, a shared understanding, shared norms, shared values, trust, cooperation, and reciprocity. Thus, option (d) is the right answer.This question is not covered in PrepMate sources.11. The Service Area Approach was implemented under the purview of(a) Integrated Rural Programme(b) Lead Bank Scheme(c) Mahatma Gandhi National Rural Employment Guarantee Scheme(d) National Skill Development MissionSol. 11 (b) Lead Bank SchemeSource: Economics book, Chapter 11, Page 17512. With reference to the management of minor minerals in India, consider the following statements:1. Sand is a 'minor mineral' according to the prevailing law in the country.2. State Governments have the power to grant mining leases of minor minerals, but the powers regarding the formation of rules related to the grant of minor minerals lie with the Central Government.3. State Governments have the power to frame rules to prevent illegal mining of minor minerals.Which of the statements given above is / are correct?(a) 1 and 3 only(b) 2 and 3 only(c) 3 only(d) 1, 2 and 3Sol. 12 (a) 1 and 3 onlyTopic: MiscellaneousStatement 1 is correct. Stand is a mirror mineral, as defined under section 3(e) of the Mines and Minerals (Development and Regulation) Act, 1957 (MMDR Act).Statement 2 is incorrect. Section 15 of the MMDR Act empowers state governments to make rules for regulating the grant of mineral concessions in respect of minor minerals. The regulation of grant of mineral concessions for minor minerals is, therefore, within the purview of the state governments.Statement 3 is correct. Section 23C of the MMDR Act, 1957 empowers state government to frame rules to prevent illegal mining, transportation and storage of minerals.13. Consider the following statements:1. Most of India's external debt is owed by governmental entities.2. All of India's external debt is denominated in US dollars.Which of the statements given above is / are correct?(a) 1 only(b) 2 only(c) Both 1 and 2(d) Neither 1 nor 2Sol. 13 (d) Neither 1 nor 2Source: PrepMate Economics Book, Chapter 23, Page 325Statement 1 is incorrect: Out of India’s total external debt, nearly 20% is owed by governmental entities.Statement 2 is incorrect. India’s External debt is denominated in multiple currencies. As on 31st December, 2017, 48.2% of debt was denominated in US dollars. The rest of debt is denominated in Indian Rupee (for instance- masala bonds), Special Drawing rights, Japanese Yen, Euro and other currencies.14. Which of the following is not included in the assets of a commercial bank in India?(a) Advances(b) Deposits(c) Investments(d) Money at call and short noticeSol. 14 (b) DepositsSource: Economics book, Chapter 3, Page 5915. In the context of India, which of the following factors is/are contributor/ contributors to reducing the risk of a currency crisis?1. The foreign currency earnings of India's IT sector2. Increasing the government expenditure3. Remittances from Indians abroadSelect the correct answer using the code given below.(a) 1 only(b) 1 and 3 only(c) 2 only(d) 1, 2 and 3Sol. 15 (b) 1 and 3 onlySource: PrepMate Economics book, Chapter 21, Page 306This question is based on understanding of basic economics and knowledge of current affairs. Currency crisis in the present context refers to depreciation of rupee. Any foreign exchange transaction which generates demand of rupee can help in arresting the depreciation of rupee.Statement 1 is correct. Foreign currency earnings of India's IT sector generate supply of dollar and demand of Indian rupee. Thus, this transaction helps in arresting the depreciation of rupee.Statement 2 is incorrect. Increase in government expenditure doesn’t directly affects the demand and supply of rupee.Statement 3 is correct. Remittances from Indians abroad generate supply of dollar and demand of Indian rupee. Thus, this transaction helps in arresting the depreciation of rupee.Thus, Statements 1 and 3 are correct.16. Which one of the following suggested that the Governor should be an eminent person from outside the State and should be a detached figure without intense political links or should not have taken part in politics in the recent past?(a) First Administrative Reforms Commission (1966)(b) Rajamannar Committee (1969)(c) Sarkaria Commission (1983)(d) National Commission to Review the Working of the Constitution (2000)Sol. 16 (c) Sarkaria Commission (1983)Source: PrepMate Current AffairsThere have been instances in recent past when Governors have acted in the interest of central government. It is usual for news articles to recall the recommendations of Sarkaria commission on office of governor at such instances. There was one article in Indian Express which was shared in PrepMate newsjuice. Relevant extracts of that article are pasted below.17. Which of the following is issued by registered foreign portfolio investors to overseas investors who want to be part of the Indian stock market without registering themselves directly?(a) Certificate of Deposit(b) Commercial Paper(c) Promissory Note(d) Participatory NoteSol. 17 (d) Participatory NoteSource: PrepMate Economics, Chapter 24, Page 343There is a direct question in practice set given at end of chapter which is similar to this question. A participatory note, commonly known as a P-note or PN, is a certificate issued by a registered foreign institutional investor (FII) to an overseas investor who invests in Indian stock markets. FII is registered with SEBI directly, but overseas investor is not required to register themselves with SEBI. Thus, overseas investor invests in Indian stock market through a registered foreign institutional investor. As proof of investment, a certificate is issued by FII to the overseas investor.18. Consider the following statements:1. As per law, the Compensatory Afforestation Fund Management and Planning Authority exists at both National and State levels.2. People's participation is mandatory in the compensatory afforestation programmes carried out under the Compensatory Afforestation Fund Act, 2016.Which of the statements given above is / are correct?(a) 1 only(b) 2 only(c) Both 1 and 2(d) Neither 1 nor 2Sol. 18 (a) 1 onlySource: PrepMate Current AffairsStatement 1 is correct.Statement 2 is incorrect. Compensatory Afforestation Fund Act, 2016 was criticized on the grounds that it does not provide for people’s participation in afforestation programmes.19. In India, which of the following review the independent regulators in sectors like telecommunications, insurance, electricity, etc.?1. Ad Hoc Committees set up by the Parliament2. Parliamentary Department Related Standing Committees3. Finance Commission4. Financial Sector Legislative Reforms Commission5. NITI AayogSelect the correct answer using the code given below.(a) 1 and 2(b) 1, 3 and 4(c) 3, 4 and 5(d) 2 and 5Sol.19 (a) 1 and 2Source: PrepMate Polity book, Chapter 8, Page 143 and 144This is an indirect question. This question requires understanding about the functioning of the mentioned bodies.Statement 1 is correct: Ad Hoc Committees of parliament can be established to advice and enquire on particular matter. Thus, they can be established to review the independent regulators.Statement 2 is correct: Parliamentary Department Related Standing Committees undertake review of functioning, accounts and bills of particular ministry/ department. Thus, they can also review the independent regulator pertaining to that particular department.Statement 3 is incorrect: Finance commission recommends division of funds between Centre and States. It does not review the independent regulators.Statement 4 is incorrect: Financial Sector Legislative Reforms Commission make recommendations to bring reforms in financial sector.Statement 5 is incorrect: Niti Aayog has undertaken various tasks such as adoption of best practices in government machinery, oversee performance of certain tasks, recommendations on certain issues, etc. However, it is not involved in review of independent regulators.20. With reference to India's Five-Year Plans, which of the following statements is/are correct?1. From the Second Five-Year Plan, there was a determined thrust towards substitution of basic and capital good industries.2. The Fourth Five-Year Plan adopted the objective of correcting the earlier trend of increased concentration of wealth and economic power.3. In the Fifth Five-Year Plan, for the first time, the financial sector was included as an integral part of the Plan.Select the correct answer using the code given below.(a) 1 and 2 only(b) 2 only(c) 3 only(d) 1, 2 and 3Sol. 20 (a) 1 and 2 onlySource: PrepMate Economics, Chapter 13, Page 192This question requires not just understanding of Planning and but also keen analysis.Statement 1 is correct: The thrust towards substitution of basic and capital good industries is from second five year plan after adoption of Mahalanobis model.Statement 2 is correct: In order to correct the trend of increased concentration of wealth and economic power, government carried out nationalization of important industries such as banking, insurance, etc.Statement 3 is incorrect: Financial sector includes industries such as banking, insurance which were included from the first plan itself.21. With reference to the Constitution of India, consider the following statements:1. No High Court shall have the jurisdiction to declare any central law to be constitutionally invalid.2. An amendment to the Constitution of India cannot be called into question by the Supreme Court of India.Which of the statements given above is / are correct?(a) 1 only(b) 2 only(c) Both 1 and 2(d) Neither 1 nor 2Sol. 21 (d) Neither 1 nor 2This question can be answered by reading either PrepMate Indian Polity, Chapter 5, Page 43 or PrepMate Current affairs.Statement 1 is incorrect: The Indian Constitution has established an integrated judicial system with the Supreme Court at the top and the state high courts below it. This single system of courts enforces and reviews both the Central laws as well as the state laws. Thus, Courts including High Courts have the jurisdiction to declare any central law to be constitutionally invalid.Alternately, there have been many instances in recent times when high courts have declared central law as constitutionally invalid.Statement 2 is also incorrect: Supreme Court held in the Kesavananda Bharati case (1973) that a Constitutional amendment can be challenged on the ground that it violates the ‘basic structure’ of the Constitution and hence, can be declared as void.Source: PrepMate Polity, Chapter 5, Page 43Even recently, Supreme Court has held 99th Constitutional amendment act which provided for NJAC as unconstitutional and void.Source: PrepMate Polity, Chapter 8, Page 15022. Consider the following statements:1. Purchasing Power Parity (PPP) exchange rates are calculated by comparing the prices of the same basket of goods and services in different countries.2. In terms of PPP dollars, India is the sixth largest economy in the world.Which of the statements given above is / are correct?(a) 1 only(b) 2 only(c) Both 1 and 2(d) Neither 1 nor 2Sol. 22 (a) 1 onlySource: PrepMate Economics, Chapter 1, Page 223. With reference to the cultivation of Kharif crops in India in the last five years, consider the following statements:1. Area under rice cultivation is the highest.2. Area under the cultivation of jowar is more than that of oilseeds.3. Area of cotton cultivation is more than that of sugarcane.4. Area under sugarcane cultivation has steadily decreased.Which of the statements given above are correct?(a) 1 and 3 only(b) 2, 3 and 4 only(c) 2 and 4 only(d) 1, 2, 3 and 4Sol. 23 (a) 1 and 3 onlySource: Geography book, Chapter 27, Page 357This question can be solved easily through combination of selection and elimination techniques. The question stem (With reference to the cultivation of Kharif crops in India in the last five years, consider the following statements) makes it even easier to solve this question. The question talks about only kharif crops.Let us start from Statement 1. It is clear that amongst various kharif crops, the largest area is under rice cultivation.Once we ascertain that Statement 1 is correct, we are left with answer options (a) 1 and 3 only and(d) 1, 2, 3 and 4.Now, we can safely rule out Statement 4. There is no information that area under sugarcane cultivation has steadily decreased. It would have been a big news that area under sugarcane cultivation is continuously decreasing.Thus, we can safely eliminate option (d). Our answer is remaining option, which is option (a).There is another interesting statement in this question, Statement 3- Area of cotton cultivation is more than that of sugarcane. Sugarcane is cultivated both as kharif and rabi crop. On the other hand, cotton is cultivated only as kharif crop. Certainly, when we consider this fact, then for sure area of cotton cultivation is more than that of sugarcane.24. Among the agricultural commodities imported by India, which one of the following accounts for the highest imports in terms of value in the last five years?(a) Spices(b) Fresh fruits(c) Pulses(d) Vegetable oilsSol. 24 (d) Vegetable oilsSource: PrepMate Economics, Chapter 27, Page 382Among the agricultural commodities imported by India, Vegetable oils constitute significant chunk of imports.25. In the context of polity, which one of the following would you accept as the most appropriate definition of liberty?(a) Protection against the tyranny of political rulers(b) Absence of restraint(c) Opportunity to do whatever one likes(d) Opportunity to develop oneself fullySol. 25 (d) Opportunity to develop oneself fullyTopic: PolityThere has been question on concept of liberty in prelims 2017 and 2018 as well. PrepMate Polity book contains detailed explanations to these questions. These explanations and understanding of fundamental rights are useful to solve the above question. However, we don’t claim this question to be appearing from our study material because this question demands deeper theoretical knowledge of concepts. Let us examine the answer options.Option (a) is incorrect: It fails to express the complete idea of liberty. Protection against the tyranny of political rulers does not express complete idea of liberty. Liberty not only confers protection but it also grants multifaceted freedoms.Option (b) is incorrect: If there is no restraint, then the actions of people may erode other’s liberty. Thus, restraint is essential to enjoy liberty. For instance, even fundamental rights have restrictions. There are 8 limitations on freedom of speech and expression.Option (c) is incorrect: It also attempts to grant freedom without any restraint. The explanation given for proving option (b) as incorrect, is also applicable for option (c).Option (d) is correct: Liberty (with restraints) provide us with opportunity to develop fully in life. The development may be in different dimensions such as social, economic, intellectual, etc. Let us look at the other side. If there were severe restrictions by state on economic activity, then we may not be able to pursue our economic upliftment. It is to be noted that a developed individual has utmost respect for restraints because one’s restraint enable other people to enjoy their own liberty.26. Which one of the following is not the most likely measure the Government/RBI takes to stop the slide of Indian rupee?(a) Curbing imports of non-essential goods and promoting exports(b) Encouraging Indian borrowers to issue rupee denominated Masala Bonds(c) Easing conditions relating to external commercial borrowing(d) Following an expansionary monetary policySol. 26 (d) Following an expansionary monetary policySource: PrepMate Economics book, Chapter 21, Page 306 and Chapter 24, Page 345Options (a), (b) and (c) are adopted to curb the slide of Indian rupee.Chapter 21, Page 306Option (c) is not the answer. Easing conditions relating to external commercial borrowing will encourage fresh loans from external sources.Option (b) is not the answer.Source PrepMate Economics book, Chapter 24, Page 345Option (d) is the answer. Option (d) will not help in curbing the slide of Indian rupee. Rather, it may even further reduce the value of Indian rupee. If expansionary monetary policy is followed, it leads to increase in money supply. Increase in money supply may lead to inflation, which may further lead to fall in value of rupee.27. Consider the following statements:The Reserve Bank of India's recent directives relating to 'Storage of Payment System Data', popularly known as data diktat, command the payment system providers that1. they shall ensure that entire data relating to payment systems operated by them are stored in a system only in India2. they shall ensure that the systems are owned and operated by public sector enterprises3. they shall submit the consolidated system audit report to the Comptroller and Auditor General of India by the end of the calendar yearWhich of the statements given above is/are correct?(a) 1 only(b) 1 and 2 only(c) 3 only(d) 1, 2 and 3Sol. 27 (a) 1 onlyTopic: Current AffairsStatement 1 is correct: It is important to have unfettered supervisory access to data relating to payment systems operating in India. This access is only possible if data is stored in India. This directive was released by the RBI.RBI directive was as follows. All system providers shall ensure that the entire data relating to payment systems operated by them are stored in a system only in India. This data should include the full end-to-end transaction details / information collected / carried / processed as part of the message / payment instruction. For the foreign leg of the transaction, if any, the data can also be stored in the foreign country, if required.Statement 2 is not correct: There was no directive related to the ownership and operation of the systems by public sector enterprises.Statement 3 is not correct: System providers shall submit the System Audit Report (SAR) conducted by CERT-IN empaneled auditors. The SAR duly approved by the Board of the system providers should be submitted to the Reserve Bank by the end of calendar year.28. Which of the following adopted a law on data protection and privacy for its citizens known as 'General Data Protection Regulation' in April 2016 and started implementation of it from 25th May, 2018?(a) Australia(b) Canada(c) The European Union(d) The United States of AmericaSol. 28 (c) The European UnionSource: PrepMate Current Affairs29. Recently, India signed a deal known as 'Action Plan for Prioritization and Implementation of Cooperation Areas in the Nuclear Field' with which of the following countries?(a) Japan(b) Russia(c) The United Kingdom(d) The United States of AmericaSol. 29(b) RussiaTopic: Current AffairsThis is a tough question. The deal asked in the question is related to action plan only and not the actual agreement on cooperation in nuclear field. Usually, there are many such bilateral deals with other nations. They are in nature of Memorandum of agreement and not the actual agreement.Action Plan for Prioritization and Implementation of Co-operation Areas in the Nuclear Field Identified Jointly by India and Russia was signed by India with Russia. It was signed on 5th October, 2018 in New Delhi during the visit of Vladimir Putin, President of the Russian Federation to India.30. The money multiplier in an economy increases with which one of the following?(a) Increase in the cash reserve ratio(b) Increase in the banking habit of the population(c) Increase in the statutory liquidity ratio(d) Increase in the population of the countrySol. 30 (b) Increase in the banking habit of the populationSource: PrepMate Economics, Chapter 3, Page 61Options (a) and (c) are incorrect. If cash reserve ratio and statutory liquidity ratio reduce (and not increase), then money multiplier increases. Reserve money, which is denominator for calculating money multiplier, includes statutory reserves of banks held with themselves and cash reserves of banks held with the RBI. The lower the value of denominator in a fraction, the higher the value of overall fraction.Option (d) is not directly related to the question.Option (c) is the correct answer. M1 and M3, which are numerator for calculating money multiplier, include demand deposits and total deposits by public. The amount of deposits made by people in banks also depends upon the banking habit of the population. Moreover, greater the value of numerator, greater is the value of overall fraction. Thus, increase in the banking habit of the population, increases the money multiplier in the economy.31. Consider the following statements about Particularly Vulnerable Tribal Groups (PVTGs) in India:1. PVTGs reside in 18 States and one Union Territory.2. A stagnant or declining population is one of the criteria for determining PVTG status.3. There are 95 PVTGs officially notified in the country so far.4. Irular and Konda Reddi tribes are included in the list of PVTGs.Which of the statements given above are correct?(a) 1, 2 and 3(b) 2, 3 and 4(c) 1, 2 and 4(d) 1, 3 and 4Sol. 31 (c) 1, 2 and 4Source: PrepMate Geography Book, Chapter 30, Page 401Statement 3 is incorrect. Once we rule out statement 3, answer is obvious.32. With reference to the Constitution of India, prohibitions or limitations or provisions contained in ordinary laws cannot act as prohibitions or limitations on the constitutional powers under Article 142. It could mean which one of the following?(a) The decisions taken by the Election Commission of India while discharging its duties cannot be challenged in any court of law.(b) The Supreme Court of India is not constrained in the exercise of its powers by laws made by the Parliament.(c) In the event of grave financial crisis in the country, the President of India can declare Financial Emergency without the counsel from the Cabinet.(d) State Legislatures cannot make laws on certain matters without the concurrence of Union Legislature.Sol. 32 (b) The Supreme Court of India is not constrained in the exercise of its powers by laws made by the Parliament.Source: PrepMate Polity, Chapter 8, Page 153This is an interesting question. This question requires combined analysis of Article 142 and information given in question stem. Article 142 grants extraordinary powers to the Supreme Court to do complete justice. For exercising powers conferred by Article 142, the SC is not bound by any law made by Parliament.33. With reference to the Legislative Assembly of a State in India, consider the following statements:1. The Governor makes a customary address to Members of the House at the commencement of the first session of the year.2. When a State Legislature does' not have a rule on a particular matter, it follows the Lok Sabha rule on that matter.Which of the statements given above is / are correct?(a) 1 only(b) 2 only(c) Both 1 and 2(d) Neither 1 nor 2Sol. 33 (a) 1 onlySource of 1st statement: PrepMate Polity, Chapter 8, Page 128 and Chapter 9, Page 173Statement 2 is incorrect. Article 208 Rules of procedure(1) A House of the Legislature of a State may make rules for regulating subject to the provisions of this Constitution, its procedure and the conduct of its business(2) Until rules are made under clause (1), the rules of procedure and standing orders in force immediately before the commencement of this Constitution with respect to the Legislature for the corresponding Province shall have effect in relation to the Legislature of the State subject to such modifications and adaptations as may be made therein by the Speaker of the Legislative Assembly, or the Chairman of the Legislative Council, as the case may be.34. Consider the following statements:1. The United Nations Convention against Corruption (UNCAC) has a 'Protocol against the Smuggling of Migrants by Land, Sea and Air'.2. The UNCAC is the ever-first legally binding global anti-corruption instrument.3. A highlight of the United Nations Convention against Transnational Organized Crime (UNTOC) is the inclusion of a specific chapter aimed at returning assets to their rightful owners from whom they had been taken illicitly.4. The United Nations Office on Drugs and Crime (UNODC) is mandated by its member States to assist in the implementation of both UNCAC and UNTOC.Which of the statements given above are correct?(a) 1 and 3 only(b) 2, 3 and 4 only(c) 2 and 4 only(d) 1, 2, 3 and 4Sol. 34 (c) 2 and 4 onlySource: PrepMate International Organizations and Bilateral Relations, Chapter 1, Page 15Statement 4 is correct.Statement 1 is incorrect. United Nations Convention against Corruption (UNCAC) is against corruption and is not related to Smuggling of Migrants. UNCAC has been in news in context of amendments in Prevention of Corruption Act to make it in line with UNCAC. UNTOC has a 'Protocol against the Smuggling of Migrants by Land, Sea and Air'.Once we know that statement 1 is incorrect and statement 4 is correct, we are left with options (b) and (c).Statement 3 is also incorrect. UNTOC has three supplementary protocols namely Trafficking in Persons, especially Women and Children; Smuggling of Migrants; and illicit Manufacturing of and Trafficking in Firearms. It does not have specific chapter aimed at returning assets to their rightful owners from whom they had been taken illicitly.35. Consider the following statements:1. As per recent amendment to the Indian Forest Act, 1927, forest dwellers have the right to fell the bamboos grown on forest areas.2. As per the Scheduled Tribes and Other Traditional Forest Dwellers (Recognition of Forest Rights) Act, 2006, bamboo is a minor forest produce.3. The Scheduled Tribes and Other Traditional Forest Dwellers (Recognition of Forest Rights) Act, 2006 allows ownership of minor forest produce to forest dwellers.Which of the statements given above is / are correct?(a) 1 and 2 only(b) 2 and 3 only(c) 3 only(d) 1, 2 and 3Sol. 35 (b) 2 and 3 onlySource: PrepMate Current Affairs have covered Statements 1 and 3 but not Statement 2.Statement 1 is incorrect. The Indian Forest Act, 1927, was amended to permit felling of bamboo grown in non-forest areas.Once we identify that Statement 1 is incorrect. We are left with two options- (b) 2 and 3 only and (c) 3 only.Statement 3 is correct: The Scheduled Tribes and Other Traditional Forest Dwellers (Recognition of Forest Rights) Act, 2006 provides ownership of minor forest produce to forest dwellers.Statement 2 is correct. According to the Act, ""minor forest produce"" includes all non-timber forest produce of plant origin including bamboo, brush wood, stumps, cane, tussar, cocoons, honey, wax, lac, tendu or kendu leaves, medicinal plants and herbs, roots, tubers and the like.36. Which Article of the Constitution of India safeguards one's right to marry the person of one's choice?(a) Article 19(b) Article 21(c) Article 25(d) Article 29Sol. 36 (b) Article 21Source: PrepMate Current AffairsRecently, Supreme Court has in two different cases held that one's right to marry the person of one's choice comes within the purview of Article 21. These cases relate to (1) Decision of a hindu girl in Kerala named Hadiya to marry muslim man of his choice and (2) Matter of honour killing.37. Consider the following statements:1. According to the Indian Patents Act, a biological process to create a seed can be patented in India.2. In India, there is no Intellectual Property Appellate Board.3. Plant varieties are not eligible to be patented in India.Which of the statements given above is/are correct?(a) 1 and 3 only(b) 2 and 3 only(c) 3 only(d) 1, 2 and 3Sol. 37 (c) 3 onlyTopic: EconomicsAll the answer options have Statement 3. Thus, there is no need to evaluate Statement 3.Statement 2 is incorrect: The Intellectual Property Appellate Board (IPAB) was constituted in 2003 to hear appeals against the decisions of the registrar under the Indian Trademarks Act, 1999 and the Geographical Indications of Goods (Registration and Protection) Act, 1999. IPAB is time and again mentioned in current affairs.Statement 1 is incorrect: Plants and animals in whole or any part thereof including seeds, varieties and species and biological processes for production or propagation of plants and animals are not patentable in India.38. Consider the following statements:The Environment Protection Act, 1986 empowers the Government of India to1. state the requirement of public participation in the process of environmental protection, and the procedure and manner in which it is sought2. lay down the standards for emission or discharge of environmental pollutants from various sourcesWhich of the statements given above is/ are correct?(a) 1 only(b) 2 only(c) Both 1 and 2(d) Neither 1 nor 2Sol. 38 (b) 2 onlyTopic: EnvironmentStatement 2 is correct. The Environment Protection Act, 1986 empowers the Government of India to lay down the standards for emission or discharge of environmental pollutants from various sources.Statement 1 is incorrect. There is no provision in the act which empowers the Government of India to state the requirement of public participation in the process of environmental protection, and the procedure and manner in which it is sought.39. As per the Solid Waste Management Rules, 2016 in India, which one of the following statements is correct?(a) Waste generator has to segregate waste into five categories.(b) The Rules are applicable to notified urban local bodies, notified towns and all industrial townships only.(c) The Rules provide for exact and elaborate criteria for the identification of sites for landfills and waste processing facilities.(d) It is mandatory on the part of waste generator that the waste generated in one district cannot be moved to another district.Sol. 39 (c) The Rules provide for exact and elaborate criteria for the identification of sites for landfills and waste processing facilities.Topic: Current AffairsSolid waste management rules, 2016 have been regularly in news.Option (a) is incorrect: Waste generators would now have to now segregate waste into three categories (not five) – Biodegradables, Dry (Plastic, Paper, metal, wood, etc.) and Domestic Hazardous waste (diapers, napkins, mosquito repellants, cleaning agents etc.) before handling it over to the collector.Option (b) is incorrect: The Rules are applicable beyond notified urban local bodies, notified towns and all industrial townships, and will extend to all other urban agglomerations such as census towns, notified industrial townships, areas under the control of Indian Railways,Option (c): is correct: The landfills shall be 100 meters away from the river; 200 meters from a pond; 500 meters away from the highway, habitations, public parks and water supply wells and 20 km away from airports/airbase. The construction of landfills on hills shall be avoided. Land for construction of sanitary landfills in hilly areas shall be identified in the plain areas.Option (d) is incorrect: The bio-degradable waste should be processed, treated and disposed of within the premises of generator as far as possible. The residual waste shall be given to the waste collectors or agency as directed by the local authority. There is no such restriction that it is mandatory on the part of waste generator that the waste generated in one district cannot be moved to another district.40. Consider the following statements:As per the Industrial Employment (Standing Orders) Central (Amendment) Rules, 20181. if rules for fixed-term employment are implemented, it becomes easier for the firms/companies to layoff workers.2. no notice of termination of employment shall be necessary in the case of temporary workman.Which of the statements given above is / are correct?(a) 1 only(b) 2 only(c) Both 1 and 2(d) Neither 1 nor 2Sol. 40 (c) Both 1 and 2Topic: Current AffairsThis question demands knowledge of Industrial Employment (Standing orders) Central (Amendment) Rules, 2018. These rules were passed to give flexibility to businesses in India.Statement 1 is correct: If rules for fixed team employment are implemented, it become easier for the firms/companies to lay off workers.Statement 2 is correct: No notice of termination of employment shall be necessary in the case of temporary workman. However, there are certain conditions:·Provided that a temporary workman, who has completed three months continuous service, shall be given two weeks’ notice of the intention to terminate his employment.·Provided further that when the services of a temporary workman, who has not completed three month’s continuous service, are terminated before the completion of the term of employment given to him, he shall be informed of the reasons for termination in writing.41. In the context of digital technologies for entertainment, consider the following statements:1. In Augmented Reality (AR), a simulated environment is created and the physical world is completely shut out.2. In Virtual Reality (VR), images generated from a computer are projected onto real-life objects or surroundings.3. AR allows individuals to be present in the world and improves the experience using the camera of smart-phone or PC.4. VR closes the world, and transposes an individual, providing complete immersion experience.Which of the statements given above is / are correct?(a) 1 and 2 only(b) 3 and 4(c) 1, 2 and 3(d) 4 onlySol. 41 (b) 3 and 4Topic: Science & TechnologyStatement 1 is incorrect and Statement 3 is correct: Augmented reality (AR) adds digital elements to a live view often by using the camera on a smartphone. Examples of augmented reality experiences include Snapchat lenses and games such as Pokemon Go.Statement 2 is incorrect and Statement 4 is correct: Virtual reality (VR) refer to complete immersion experience that shuts out the physical world. Using VR devices users can be transported to imagined environments such as among the dinosaurs or within a haunted house.42. The word 'Denisovan' is sometimes mentioned in media in reference to(a) fossils of a kind of dinosaurs(b) an early human species(c) a cave system found in North-East India.(d) a geological period in the history of Indian subcontinentSol. 42 (b) an early human speciesSource: Current AffairsDenisovan is an early human species which was adapted to low-oxygen environment. The first evidence for Denisovans or Denisova hominins was first discovered in 2008 in a cave in the Altai mountains in Siberia.Recently, a fossil jawbone containing molars recovered from Baishiya Karst cave in Xiahe, Gansu, China shows Denisovans lived in the Tibetan Plateau some 1,60,000 years ago.43. With reference to the recent developments in science, which one of the following statements is not correct?(a) Functional chromosomes can be created by joining segments of DNA taken from cells of different species.(b) Pieces of artificial functional DNA can be created in Iaboratories.(c) A piece of DNA taken out from an animal cell can be made to replicate outside a living cell in a laboratory.(d) Cells taken out from plants and animals can be made to undergo cell division in laboratory petri dishes.Sol. 43 (a) Functional chromosomes can be created by joining segments of DNA taken from cells of different species.Source: PrepMate Science & TechnologyThis question can be solved through elimination technique.Let us start from option (d). It is easiest. The statement given in answer option can be eliminated on the basis of cloning technique. In the process of cloning, cells taken out from plants and animals undergo cell division in laboratory petri dishes.Source: PrepMate Science & Technology, Chapter 1, Page 8Options (b) and (c) can also be eliminated. Synthetic biology is used to create pieces of artificial functional DNA. It can also be used to replicate DNA outside a living cell.Source: PrepMate Science & Technology, Chapter 1, Page 744. Consider the following statements:A digital signature is1. an electronic record that identifies the certifying authority issuing it2. used to serve as a proof of identity of an individual to access information or server on Internet.3. an electronic method of signing an electronic document and ensuring that the original content is unchangedWhich of the statements given above is / are correct?(a) 1 only(b) 2 and 3 only(c) 3 only(d) 1, 2 and 3Sol. 44 (d) 1, 2 and 3Source: PrepMate Science & Technology, Chapter 11, Page 159Statements 1 and 3 are correct.Statement 2 is also correct. A digital certificate can also be presented electronically to prove one’s identity, to access information or services on the Internet.45. In the context of wearable technology, which of the following tasks is/are accomplished by wearable devices?1. Location identification of a person2. Sleep monitoring of a person3. Assisting the hearing impaired personSelect the correct answer using the code given below.(a) 1 only(b) 2 and 3 only(c) 3 only(d) 1, 2 and 3Sol.45 (d) 1, 2 and 3Source: PrepMate Current AffairsWearable technology is a category of electronic devices that can be worn as accessories, embedded in clothing, implanted in the user's body, or even tattooed on the skin.This question can be inferred on the basis of article on Apple watch in Indian express, which was covered in PrepMate current affairs.46. 'RNA interference (RNAi)' technology has gained popularity in the last few years. Why?1. It is used in developing gene silencing therapies.2. It can be used in developing therapies for-the treatment of cancer.3. It can be used to develop hormone replacement therapies.4. It can be used to produce crop plants that are resistant to viral pathogens.Select the correct answer using the code given below.(a) 1, 2 and 4(b) 2 and 3(c) 1 and 3(d) 1 and 4 onlySol. 46 (a) 1, 2 and 4Source: PrepMate General Science Book, Biology Section, Chapter 4, Page 233This question can be solved by understanding functioning of RNA.Statement 1 is correct. As RNA transcribes genetic information from the DNA found in a cell’s nucleus and then carries this information to other cell organelles, RNAi is used in developing gene silencing therapies.Statement 2 is correct. Cancer is abnormal growth of body cells which is related to DNA present in them. As DNA are expressed through RNA, RNAi can be used in developing therapies for-the treatment of cancer.Statement 3 is incorrect. So far, genetic information has not been intrinsically related to our hormone system. Thus, RNAi It cannot be used to develop hormone replacement therapies.Statement 4 is correct. RNA can impact expression of DNA which may make plant vulnerable to viral pathogens. Thus, RNAi can be used to produce crop plants that are resistant to viral pathogens.47. Recently, scientists observed the merger of giant 'blackholes' billions of light-years away from the Earth. What is the significance of this observation?(a) 'Higgs boson particles' were detected.(b) 'Gravitational waves' were detected.(c) Possibility of inter-galactic space travel through 'wormhole' was confirmed.(d) It enabled the scientists to understand 'singularity'.Sol. 47 (b) 'Gravitational waves' were detected.Source: PrepMate Science & Technology Book, Chapter 10, Page 14848. Which of the following are the reasons for the occurrence of multi-drug resistance in microbial pathogens in India?1. Genetic predisposition of some people2. Taking incorrect doses of antibiotics to cure diseases3. Using antibiotics in livestock farming4. Multiple chronic diseases in some peopleSelect the correct answer using the code given below.(a) 1 and 2(b) 2 and 3 only(c) 1, 3 and 4(d) 2, 3 and 4Sol. 48 (b) 2 and 3 onlySource: PrepMate Science & Technology, Chapter 15, Page 206Anything which is associated with excessive use of antibiotics is a cause behind antibiotic resistance.Statement 2 is correct. Incorrect doses of antibiotics lead to excessive use of antibiotics.Statement 3 is correct. This statement has been specifically mentioned in current affairs. Antibiotic resistance in humans has also been traced to resistant microbes originating in livestock. Thus, increased use of antibiotics in livestock also leads to antibiotic resistance.Statements 1 and 4 are not directly related to antibiotic resistance.49. What is Cas9 protein that is often mentioned in news?(a) A molecular scissors used in targeted gene editing(b) A biosensor used in the accurate detection of pathogens in patients(c) A gene that makes plants pest-resistant(d) A herbicidal substance synthesized in genetically modified cropsSol. 49 (a) A molecular scissors used in targeted gene editingSource: PrepMate Current Affairs50. Which one of the following statements is not correct?(a) Hepatitis B virus is transmitted much like HIV.(b) Hepatitis B, unlike Hepatitis C, does not have a vaccine.(c) Globally, the number of people infected with Hepatitis B and C viruses are several times more than those infected with HIV.(d) Some of those infected with Hepatitis Band C viruses do not show the symptoms for many years.Sol. 50 (b) Hepatitis B, unlike Hepatitis C, does not have a vaccine.Source: PrepMate Current AffairsThis question has been picked up from a news article in The Hindu. All the answer options are from single news article. This article was shared in PrepMate News Juice.51. With reference to Mughal India, what is/are the difference/differences between Jagirdar and Zamindar?1. Jagirdars were holders of land assignments in lieu of judicial and police duties, whereas Zamindars were holders of revenue rights without obligation to perform any duty other than revenue collection.2. Land assignments to Jagirdars were hereditary and revenue rights of Zamindars were not hereditary.Select the correct answer using the code given below.(a) 1 only(b) 2 only(c) Both 1 and 2(d) Neither 1 nor 2Sol. 51 (d) Neither 1 nor 2Source: PrepMate Ancient and Medieval History & Culture, Chapter 16, Page 217Statement 2 is incorrect: Akbar introduced mansabdari system which was based on merit. The system included all the officials included nobles which were also jagirdars. Thus, we can conclude that land assignments to jagirdars were not hereditary.Statement 1 is also incorrect. Jagirdars included people such as nobles. They were not required to perform judicial and police duties.52. With reference to land reforms in independent India, which one of the following statements is correct?(a) The ceiling laws were aimed at family holdings and not individual holdings.(b) The major aim of land reforms was providing agricultural land to all the landless.(c) It resulted in cultivation of cash crops as a predominant form of cultivation.(d) Land reforms permitted no exemptions to the ceiling limits.Sol. 52 (b) The major aim of land reforms was providing agricultural land to all the landless.Source: PrepMate Economics, Chapter 12, Page 184Option (a) is incorrect. Ceiling laws were aimed at both family and individual holdings.Option (c) is incorrect. Land reforms did not encourage cultivation of cash crops.Option (d) is incorrect. There were exceptions to land ceiling limits.Option (b) is the correct answer. The major aim of land reforms was providing agricultural land to all the landless.53. The Global Competitiveness Report is published by the(a) International Monetary Fund(b) United Nations Conference on Trade and Development(c) World Economic Forum(d) World BankSol. 53(c) World Economic ForumSource: PrepMate International Organizations and Bilateral Relations, Chapter 7, Page 16654. Consider the following statements about 'the Charter Act of 1813':1. It ended the trade monopoly of the East India Company in India except for trade in tea and trade with China.2. It asserted the sovereignty of the British Crown over the Indian territories held by the Company.3. The revenues of India were now controlled by the British Parliament.Which of the statements given above are correct?(a) 1 and 2 only(b) 2 and 3 only(c) 1 and 3 only(d) 1, 2 and 3Sol. 54 (a) 1 and 2 onlySource of 1st Statement : PrepMate Modern History Book, Chapter 7, Page 63Statement 1 is correct: Although the Act renewed the Charter for a further period of twenty years, it took away the monopoly of the Company to trade in India except trade in tea and trade with China.Statement 2 is correct The 1813 Act mentioned about sovereignty of the British Crown over the Indian territories held by the Company.Statement 3 is not correct: From 1853 Act, the revenues of India were controlled by the British Parliament.55. With reference to Swadeshi Movement, consider the following statements:1. It contributed to the revival of the indigenous artisan crafts and industries.2. The National Council of Education was established as a part of Swadeshi Movement.Which of the statements given above is/are correct?(a) 1 only(b) 2 only(c) Both 1 and 2(d) Neither 1 nor 2Sol. 55 (c) Both 1 and 2Source: PrepMate Modern History Book, Chapter 9, Page 9856. Consider the following pairs:Movement/Organization- Leader1. All India Anti-Untouchability League - Mahatma Gandhi2. All India Kisan Sabha - Swami Sahajanand Saraswati3. Self-Respect Movement - E. V. Ramaswami NaickerWhich of the pairs given above is/are correctly matched?(a) 1 only(b) 1 and 2 only(c) 2 and 3 only(d) 1, 2 and 3Sol. 56 (d) 1, 2 and 3Source: PrepMate Modern History Book; Statement 1 – Chapter 3, Page 26; Statement 2 – Chapter 4, Page 45; Statement 3 – Chapter 3, Page 28Statement 1Statement 2Statement 357. Which one of the following is not a Harappan site?(a) Chanhudaro(b) Kot Diji(c) Sohgaura(d) DesalpurSol. 57 (c) SohgauraSource: PrepMate Ancient and Medieval History & Culture, Chapter 7 answer explanations, Page 403Sohgaura is a mauryan period site.58. In which of the following relief sculpture inscriptions is 'Ranyo Ashoka' (King Ashoka) mentioned along with the stone portrait of Ashoka?(a) Kanganahalli(b) Sanchi I(c) Shahbazgarhi(d) SohgauraSol. (a) KanganahalliTopic: Ancient HistoryThe Stone Portrait of Ashoka at KanaganahalliThe inscription below the portrait written in brahmi script reads “Ranyo Ashoka” (King Ashoka).59. Consider the following:1. Deification of the Buddha2. Treading the path of Bodhisattvas3. Image worship and ritualsWhich of the above is/are the feature/ features of Mahayana Buddhism?(a) 1 only(b) 1 and 2 only(c) 2 and 3 only(d) 1, 2 and 3Sol. 59 (d) 1, 2 and 3Source: PrepMate Ancient and Medieval History & Culture, Chapter 5, Page 6160. With reference to forced labour (Vishti) in India during the Gupta period, which one of the following statements is correct?(a) It was considered a source of income for the State, a sort of tax paid by the people.(b) It was totally absent in the Madhya Pradesh and Kathiawar regions of the Gupta Empire.(c) The forced labourer was entitled to weekly wages.(d) The eldest son of the labourer was sent as the forced labourer.Sol. 60 (a) It was considered a source of income for the State, a sort of tax paid by the people.Topic: Ancient HistoryDuring Gupta period, Vishti was considered a source of income for the State, a sort of tax paid by the people.61. Which one of the following groups of plants was domesticated in the 'New World' and introduced into the 'Old World'?(a) Tobacco, cocoa and rubber(b)Tobacco, cotton and rubber.(c) Cotton, coffee and sugarcane(d) Rubber, coffee and wheatSol. 61 (a) Tobacco, cocoa and rubberTopic: World HistoryThis question is totally unexpected. Old World refers to Africa, Asia, and Europe while New World refers to the Americas, including North America, Central America, and South America. Europe, Asia, and Africa shared a common agricultural history. The Old World crops include wheat, rye, oats, lentils, and barley. Such crops did not exist in the Americas until their introduction in the 1490s by post-Columbian contact. The famous New World crops include rubber, tobacco, sunflower, cocoa, and cashew. The merchants of Europe took products of the New World to Europe, including tobacco, potatoes, cocoa and rubber.62. Consider the following statements:1. Asiatic lion is naturally found in India only.2. Double-humped camel is naturally found in India only.3. One-horned rhinoceros is naturally found in India only.Which of the statements given above is / are correct?(a) 1 only(b) 2 only(c) 1 and 3 only(d) 1, 2 and 3Sol. 62 (a) 1 onlySource: PrepMate Environment & Biodiversity book, Chapter 14, Page 233 and PrepMate Current affairsStatement 1 is correct.Statement 3 is incorrect. One-horned rhinoceros is not found only in India.63. Consider the following pairs:Famous place- River1. Pandharpur- Chandrabhaga2. Tiruchirappalli- Cauvery3. Hampi- MalaprabhaWhich of the pairs given above are correctly matched?(a) 1 and 2 only(b) 2 and 3 only(c) 1 and 3 only(d) 1, 2 and 3Sol. 63 (a) 1 and 2 onlySource: PrepMate Geography, Chapter 19, Page 239This is an interesting question. It can be solved by ascertaining validity of only Statement 3. Statement 3 is incorrect. Malaprabha rises in the Western Ghats and flows in North-West Karnataka, before joining river Krishna. On the other hand, Hampi is in eastern part of Karnataka.64. In a given year in India, official poverty lines are higher in some States than in others because(a) poverty rates vary from State to State(b) price levels vary from State to State(c) Gross State Product varies from State to State(d) quality of public distribution varies from State to StateSol. 64 (b) price levels vary from State to StateSource: PrepMate Economics Book, Chapter 1, Page 13This question requires understanding of concept of poverty line. Poverty line is based on the ability to meet expenditure to fulfill bare minimum needs. This expenditure varies from state to state because price levels vary from state to state. Price levels are taken into consider through Consumer Price Index.65. In the context of which of the following do some scientists suggest the use of cirrus cloud thinning technique and the injection of sulphate aerosol into stratosphere?(a) Creating the artificial rains in some regions(b) Reducing the frequency and intensity of tropical cyclones(c) Reducing the adverse effects of solar wind on the Earth(d) Reducing the global warmingSol. 65 (d) Reducing the global warmingSource: PrepMate Current AffairsThis question is asked from an article in Indian Express. The same article was shared in PrepMate news juice.66. In the context of which one of the following are the terms 'pyrolysis and plasma gasification' mentioned?(a) Extraction of rare earth elements(b) Natural gas extraction technologies(c) Hydrogen fuel-based automobiles(d) Waste-to-energy technologiesSol. 66 (d) Waste-to-energy technologiesSource: PrepMate Current AffairsThis question is asked from an article in PIB. The same article was shared in PrepMate news juice.67. Which of the following are in Agasthyamala Biosphere Reserve?(a) Neyyar, Peppara and Shendurney Wildlife Sanctuaries; and Kalakad Mundanthurai Tiger Reserve(b) Mudumalai, Sathyamangalam and Wayanad Wildlife Sanctuaries; and Silent Valley National Park(c) Kaundinya, Gundla Brahme-swaram and Papikonda Wildlife Sanctuaries; and Mukurthi National Park(d) Kawal and Sri Venkateswara Wildlife Sanctuaries; and Nagarjunasagar-Srisailam Tiger ReserveSol. 67 (a) Neyyar, Peppara and Shendurney Wildlife Sanctuaries; and Kalakad Mundanthurai Tiger ReserveSource: PrepMate Environment & Biodiversity Book, Chapter 13, Page 212This question can also be attempted using elimination technique. Candidate can compare the location of given national parks, wildlife sanctuaries and tiger reserves with that of the Agasthyamala biosphere reserve and eliminate wrong answer choices. It is important to note a candidate can learn location of only important national parks, wildlife sanctuaries and tiger reserves. Learning location of all the national parks, wildlife sanctuaries and tiger reserves is cumbersome and is not recommended.68. Consider the following statements:1. Some species of turtles are herbivores.2. Some species of fish are herbivores.3. Some species of marine mammals are herbivores.4. Some species of snakes are viviparous.Which of the statements given above are correct?(a) 1 and 3 only(b) 2, 3 and 4 only(c) 2 and 4 only(d) 1, 2, 3 and 4Sol. 68 (d) 1, 2, 3 and 4Source of 3rd statement: PrepMate Environment & Biodiversity book, Chapter 12, Page 200Statement 3 is correct. Dugong is a marine mammal. It survives on sea grass. Thus, we can conclude that some species of marine mammals are herbivores.Statement 2 is correct. It is a common fact that some species of fish are herbivores.Statement 1 is correct. There are some turtle species such as green turtles which are herbivores.We can answer the question by knowing the validity of just these three statements.69. Consider the following pairs:Wildlife: Naturally found in1. Blue-finned Mahseer: Cauvery River2. Irrawaddy Dolphin: Chambal River3. Rusty-spotted Cat: Eastern GhatsWhich of the pairs given above are correctly matched?(a) 1 and 2 only(b) 2 and 3 only(c) 1 and 3 only(d) 1, 2 and 3Sol. 69 (c) 1 and 3 onlySource: PrepMate Environment & Biodiversity Book, Chapter 14, Page 235We can solve this question by knowing the validity of statement 2 only.Statement 2 is incorrect. Freshwater dolphins are present in Ganga and its tributaries, not Chambal.Once we rule out statement 2, answer is option (c).70. Why is there a great concern about the 'microbeads' that are released into environment?(a) They are considered harmful to marine ecosystems.(b) They are considered to cause skin cancer in children.(c) They are small enough to be absorbed by crop plants in irrigated fields.(d) They are often found to be used as food adulterants.Sol. 70 (a) They are considered harmful to marine ecosystems.Source: PrepMate Current AffairsMany times there have been articles in newspapers on Microbeads. Here is a one article in The Hindu, which was shared in PrepMate Current Affairs.71. Building 'Kalyaana Mandapas' was a notable feature in the temple construction in the kingdom of(a) Chalukya(b) Chandela(c) Rashtrakuta(d) VijayanagaraSol. 71 (d) VijayanagaraSource: PrepMate Ancient and Medieval History & Culture, Chapter 20, Page 289This question can be attempted from both selection as well as elimination technique.Option (a) can be eliminated because Chalukyas were associated with cave temples.Option (b) can be eliminated because Chandela did not emphasise on large Mandapas in temple. Their famous creation is Kandariya Mahadeva temple in Khajurao.Option (c) can be eliminated because Rashtrakutas were also not associated with large Mandapas.Option (d) is the correct answer. 'Kalyaana Mandapas' or wedding hall was built in temples by kingdom of Vijayanagara.72. Consider the following statements:1. In the revenue administration of Delhi Sultanate, the in-charge of revenue collection was known as 'Amil'.2. The Iqta system of Sultans of Delhi was an ancient indigenous institution.3. The office of 'Mir Bakshi' came into existence during the reign of Khalji Sultans of Delhi.Which of the statements given above is/are correct?(a) 1 only(b) 1 and 2 only(c) 3 only(d) 1, 2 and 3Sol. 72 (a) 1 onlySource: PrepMate Ancient and Medieval History & Culture, Chapter 13, Page 181 and Chapter 16, Page 217Statement 2 is incorrect.Statement 3 is also incorrect. The office of Mir Bakshi came into existence during reign of Mughals.73. Consider the following statements:1. Saint Nimbarka was a contemporary of Akbar.2. Saint Kabir was greatly influenced by Shaikh Ahmad Sirhindi.Which of the statements given above is/are correct?(a) 1 only(b) 2 only(c) Both 1 and 2(d) Neither 1 nor 2Sol. 73 (d) Neither 1 nor 2Source: PrepMate Ancient and Medieval History & Culture, Chapter 14, Page 190 and 192Statement 1 is incorrect. Saint Nimbarka (11th century) was not contemporary of Akbar (1542-1605).Statement 2 is incorrect. Saint kabir lived earlier than Shaikh Ahmad Sirhindi.74. With reference to the British colonial rule in India, consider the following statements:1. Mahatma Gandhi was instrumental in the abolition of the system of 'indentured labour'.2. In Lord Chelmsford's 'War Conference', Mahatma Gandhi did not support the resolution on recruiting Indians for World War.3. Consequent upon the breaking of Salt Law by Indian people, the Indian National Congress was declared illegal by the colonial rulers.Which of the statements given above are correct?(a) 1 and 2 only(b) 1 and 3 only(c) 2 and 3 only(d) 1, 2 and 3Sol. 74(b) 1 and 3 onlySource of 2nd and 3rd statement: PrepMate Modern History Book, Chapter 9, Page 102 and Chapter 14, Page 174Statement 1 is correct. Mahatma Gandhi pressurized authorities to abolish indentured labour in India. Due to this pressure, indentured labour was abolished in the year 1917.Statement 2 is incorrect. Mahatma Gandhi supported British World war I efforts. Rather, he was conferred with Kaiser-e-hind to support british war efforts.Chapter 9, Page 102Statement 3 is correct.Chapter 14, Page 17475. With reference to Indian National Movement, consider the following pairs:Person- Position held1. Sir Tej Bahadur Sapru- President, All India Liberal Federation2. K. C. Neogy- Member, The Constituent Assembly3. P. C. Joshi- General Secretary, Communist Party of IndiaWhich of the pairs given above is/are correctly matched?(a) 1 only(b) 1 and 2 only(c) 3 only(d) 1, 2 and 3Sol. 75 (d) 1, 2 and 3Source of 1st Statement: PrepMate Modern History Book, Chapter 15, Page 202Statement 1 is correct.Statement 3 is correct. P. C. Joshi was General Secretary, Communist Party of India.Once we know that Statements 1 and 3 are correct, answer is option (d).76. With reference to Mian Tansen, which one of the following statements is not correct?(a) Tansen was the title given to him by Emperor Akbar.(b) Tansen composed Dhrupads on Hindu gods and goddesses.(c) Tansen composed songs on his patrons.(d) Tansen invented many Ragas.Sol. 76 (a) Tansen was the title given to him by Emperor Akbar.Topic: Medieval HistoryOption (a) is the answer. The actual name of Tansen was Ram Tanu Pandey. He was given title of ‘Tansen’ by Raja Man Singh Tomar of Gwalior. Emperor Akbar gave him title of ‘Mian’.77. Who among the following Mughal Emperors shifted emphasis from illustrated manuscripts to album and individual portrait?(a) Humayun(b) Akbar(c) Jahangir(d) Shah JahanSol. 77 (c) JahangirSource: PrepMate Ancient and Medieval History & Culture, Chapter 21, Page 321The screenshot below taken from PrepMate Ancient and Medieval History & Culture, Chapter 21, Page 320 prove that illustrated manuscripts were popular during the reign of Akbar.78. Which one of the following National Parks lies completely in the temperate alpine zone?(a) Manas National Park(b) Namdapha National Park(c) Neora Valley National Park(d) Valley of Flowers National ParkSol. 78 (d) Valley of Flowers National ParkSource: PrepMate Geography, Chapter 25, Page 314The question stem asks about national park which lies completely in the temperate alpine zone. Thus, the national park should be completely based in higher Himalayas.79. Atal Innovation Mission is set up under the(a) Department of Science and Technology(b) Ministry of Employment(c) NITI Aayog(d) Ministry of Skill Development and EntrepreneurshipSol. 79 (c) NITI AayogSource: PrepMate Current Affairs80. On 21st June, the Sun(a) does not set below the horizon at the Arctic Circle(b) does not set below the horizon at Antarctic Circle(c) shines vertically overhead at noon on the Equator(d) shines vertically overhead at the Tropic of CapricornSol. 80 (a) does not set below the horizon at the Arctic CircleSource: PrepMate Geography, Chapter 3, Page 24On 21st June, Sun is vertically overhead at the Tropic of Cancer and there is six months of daylight on north pole. Thus, On 21st June, the Sun does not set below the horizon at the Arctic Circle.81. Consider the following statements:1. Agricultural soils release nitrogen oxides into environment.2. Cattle release ammonia into environment.3. Poultry industry releases reactive nitrogen compounds into environment.Which of the statements given above is/are correct?(a) 1 and 3 only(b) 2 and 3 only(c) 2 only(d) 1, 2 and 3Sol. 81 (b) 2 and 3Source: PrepMate Environment & Biodiversity, Chapter 2, Page 24 and Current AffairsStatement 1 is incorrect. Agricultural soils release free atmospheric nitrogen (and not nitrogen oxides) into environment.Statement 2 is correct. Ammonia is released into the environment on excretion and death of cattle.Statement 3 is correct. Poultry industry releases reactive nitrogen compounds into environment.It is interesting to note that the Statements of this question were directly picked from an article published in The Hindu.82. What is common to the places known as Aliyar, Isapur and Kangsabati?(a) Recently discovered uranium deposits(b) Tropical rain forests(c) Underground cave systems(d) Water reservoirsSol. 82 (d) Water reservoirsTopic: Current Affairs83. In the context of proposals to the use of hydrogen-enriched CNG (H-CNG) as fuel for buses in public transport, consider the following statements:1. The main advantage of the use of H-CNG is the elimination of carbon monoxide emissions.2. H-CNG as fuel reduces carbon dioxide and hydrocarbon emissions.3. Hydrogen up to one-fifth by volume can be blended with CNG as fuel for buses.4. H-CNG makes the fuel less expensive than CNG.Which of the statements given above is / are correct?(a) 1 only(b) 2 and 3 only(c) 4 only(d) 1, 2, 3 and 4Sol. 83 (b) 2 and 3 onlySource: PrepMate Current AffairsThis question is directly picked up from an article in Indian Express. This article was shared in PrepMate News Juice.Statement 1 is incorrect. H-CNG reduce carbon monoxide emissions by upto 70 %, but does not eliminate them.Statement 2 is correct: When compared to other fuels, H-CNG as fuel reduces carbon dioxide and hydrocarbon emissions.Statement 3 is correct: The appropriate Hydrogen concentration, which is 18-20% (up to one-fifth by volume) is blended with CNG.Statement 4 is incorrect: Presently, H-CNG is costlier than CNG.84. Why are dewdrops not formed on a cloudy night?(a) Clouds absorb the radiation released from the Earth's surface.(b) Clouds reflect back the Earth's radiation.(c) The Earth's surface would have low temperature on cloudy nights.(d) Clouds deflect the blowing wind to ground level.Sol. 84 (b) Clouds reflect back the Earth's radiation.Source: PrepMate Geography, Chapter 12, Page 113This question requires precise application of theoretical concepts. We understand that dew are tiny droplets of water which develop on surfaces which radiate heat and experience condensation. The faster the ability to radiate heat, the higher will be the rate of condensation and dew formation.Thus, any factor which increases the rate of heat radiation, will aid in formation of dew drops. On the other hand, any factor which reduces the rate of heat radiation, will prevent formation of dew drops.Option (b) is the correct answer because when clouds reflect back the Earth's radiation, more heat accumulates at the surface of earth. Accumulation of more heat at surface of earth prevents condensation and thus, dew formation.85. Consider the following statements:1. The 44th Amendment to the Constitution of India introduced an Article placing the election of the Prime Minister beyond judicial review.2. The Supreme Court of India struck down the 99th Amendment to the Constitution of India as being violative of the independence of judiciary.Which of the statements given above is/are correct?(a) 1 only(b) 2 only(c) Both 1 and 2(d) Neither 1 nor 2Sol. 85 (b) 2 onlySource: PrepMate Polity, Chapter 25, Page 283 and Chapter 8, Page 150Statement 1 is incorrect. PrepMate Polity, Chapter 25, Page 283Statement 2 is correct. PrepMate Polity, Chapter 8, Page 15086. Consider the following statements:1. The motion to impeach a Judge of the Supreme Court of India cannot be rejected by the Speaker of the Lok Sabha as per the Judges (Inquiry) Act, 1968.2. The Constitution of India defines and gives details of what Constitutes 'incapacity and proved misbehaviour' of the Judges of the Supreme Court of India.3. The details of the process of impeachment of the Judges of the Supreme Court of India are given in the Judges (Inquiry) Act, 1968.4. If the motion for the impeachment of a Judge is taken up for voting, the law requires the motion to be backed by each House of the Parliament and supported by a majority of total membership of that House and by not less than two-thirds of total members of that House present and voting.Which of the statements given above is/are correct?(a) 1 and 2(b) 3 only(c) 3 and 4 only(d) 1, 3 and 4Sol. 86 (c) 3 and 4 onlySource for 2nd and 4th Statement: PrepMate Polity, Chapter 8, Page 151Statement 2 is incorrect. The Constitution of India does not define what constitutes incapacity and proved misbehavior of the Judges of Supreme Court. It is for the Parliament to decide what constitutes incapacity and proved misbehavior.Statement 4 is correct. If the motion for the impeachment of a Judge is taken up for voting, the law requires the motion to be backed by each House of the Parliament and supported by a majority of total membership of that House and by not less than two-thirds of total members of that House present and voting.Statement 3 is correct. The details of the process of impeachment of the Judges of the Supreme Court of India are given in the Judges (Inquiry) Act, 1968.Statement 1 is incorrect. The motion to impeach a Judge of the Supreme Court of India can be rejected by the Speaker of the Lok Sabha as per the Judges (Inquiry) Act, 1968.87. The Ninth Schedule was introduced in the Constitution of India during the prime ministership of(a) Jawaharlal Nehru(b) Lal Bahadur Shastri(c) Indira Gandhi(d) Morarji DesaiSol.87 (a) Jawaharlal NehruSource PrepMate Polity, Chapter 5, Page 4488. Consider the following statements:1. Coal sector was nationalized by the Government of India under Indira Gandhi.2. Now, coal blocks are allocated on lottery basis.3. Till recently, India imported coal to meet the shortages of domestic supply, but now India is self-sufficient in coal production.Which of the statements given above is/are correct?(a) 1 only(b) 2 and 3 only(c) 3 only(d) 1, 2 and 3Sol. 88 (a) 1 onlySource: PrepMate Current AffairsStatement 3 is incorrect. India imports coal for its consumption.Statement 1 is correct.Statement 2 is incorrect. Now, Coal blocks are allocated on auction basis (and not lottery basis).89. Consider the following statements:1. The Parliament (Prevention of Disqualification) Act, 1959 exempts several posts from disqualification on the grounds of 'Office of Profit'.2. The above-mentioned Act was amended five times.3. The term 'Office of Profit' is well-defined in the Constitution of India.Which of the statements given above is/are correct?(a) 1 and 2 only(b) 3 only(c) 2 and 3 only(d) 1, 2 and 3Sol. 89 (a) 1 and 2 onlySource: PrepMate Polity, Chapter 8, Page 74This question can be solved on the basis of Statement 3 only.Statement 3 is incorrect. The term 'Office of Profit' is not defined in the Constitution of India. Supreme Court has defined this term.90. Under which Schedule of the Constitution of India can the transfer of tribal land to private parties for mining be declared null and void?(a) Third Schedule(b) Fifth Schedule(c) Ninth Schedule(d) Twelfth ScheduleSol. 90 (b) Fifth ScheduleSource: PrepMate Polity, Chapter 28, Page 300The fifth schedule grants powers to Governor of the state to ensure that the administration of Scheduled areas is in the best interest of tribals.91. Recently, there was a growing awareness in our country about the importance of Himalayan nettle (Girardinia diversifolia) because it is found to be a sustainable source of(a) anti-malarial drug(b) biodiesel(c) pulp for paper industry(d) textile fibreSol. 91 (d) textile fibreTopic: Current AffairsGirardinia diversifolia is a fibre yielding plant. It is considered as a sustainable source for textile fibre.92. For the measurement/estimation of which of the following are satellite images/remote sensing data used?1. Chlorophyll content in the vegetation of a specific location2. Greenhouse gas emissions from rice paddies of a specific location3. Land surface temperatures of a specific locationSelect the correct answer using the code given below.(a) 1 only(b) 2 and 3 only(c) 3 only(d) 1, 2 and 3Sol. 92 (d) 1, 2 and 3Source: PrepMate Science & Technology Book, Chapter 3, Page 49This question requires understanding of capability of remote sensing satellites (RSS). All the given statements in this question are correct.Statement 1 is correct. Chlorophyll content in the vegetation of a specific location can be estimated by observing composition of plants.Statement 2 is correct. Greenhouse gas emissions are tracked by RSS.Statement 3 is correct. RSS can track temperature through measurement of radiations.93. Consider the following States:1. Chhattisgarh2. Madhya Pradesh3. Maharashtra4. OdishaWith reference to the States mentioned above, in terms of percentage of forest cover to the total area of State, which one of the following is the correct ascending order?(a) 2-3-1-4(b) 2-3-4-1(c) 3-2-4-1(d) 3-2-1-4Sol. 93 (c) 3-2-4-1Source: PrepMate Environment & Biodiversity, Chapter 3, Page 38This question has been asked from State of forest report, 2017. The report is included in PrepMate Environment & Biodiversity Book.In the given states, only Chattisgarh falls in category of 33% to 75% area under forest cover. Thus, it has the maximum percentage of forest cover to the total area.In the given states, Maharashtra has least percentage of forest cover to the total area. The eastern part of Maharashtra has lower rainfall and thus it is in nature of savanna. The western part of Maharashtra is highly urbanized.94. Which of the following statements are correct about the deposits of 'methane hydrate’?1. Global warming might trigger the release of methane gas from these deposits.2. Large deposits of 'methane hydrate' are found in Arctic Tundra and under the seafloor.3. Methane in atmosphere oxidizes to carbon dioxide after a decade or two.Select the correct answer using the code given below.(a) 1 and 2 only(b) 2 and 3 only(c) 1 and 3 only(d) 1, 2 and 3Sol. 94 (d) 1, 2 and 3Source of 1st and 2nd Statement: PrepMate Environment & Biodiversity Book, Chapter 18, Page 267Statements 1 and 2 are correct.Statement 3 is also correct. When methane is anthropogenically emitted, methane is oxidized in the atmosphere a decade or two later. Once oxidized, the carbon in each methane molecule is converted to CO2, which then stays in the atmosphere as CO2 for another century or more.95. Consider the following:1. Carbon monoxide2. Methane3. Ozone4. Sulphur dioxideWhich of the above are released into atmosphere due to the burning of crop/biomass residue?(a) 1 and 2 only(b) 2, 3 and 4 only(c) 1 and 4 only(d) 1, 2, 3 and 4Sol. 95 (d) 1, 2, 3 and 4Source: PrepMate Current AffairsBurning of crop /biomass residues generate greenhouse gases (GHGs) such as carbon dioxide (CO2), carbon monoxide (CO), methane (CH4) and nitrous oxide (N2O) as well as volatile organic compounds (VOCs), ammonia (NH3), sulphur dioxide (SO2), nitrogen oxides (NOx) and particulate matter (PM).Ozone is a secondary pollutant. It does not emerges directly from burning of crop/biomass residue. However, agricultural residues burning also releases a huge amount of pollutants to the atmosphere, which apart from the above includes aerosols and hydrocarbons. Haze formation is one of the major impacts of open burning of rice fields. This haze has become an important source of tropospheric ozone and the aerosols resulting from such events have contributed largely to the formation of atmospheric brown clouds.96. Consider the following pairs:Sea: Bordering country1. Adriatic Sea: Albania2. Black Sea: Croatia3. Caspian Sea: Kazakhstan4. Mediterranean Sea: Morocco5. Red Sea: SyriaWhich of the pair given above are correctly matched?(a) 1, 2 and 4 only(b) 1, 3 and 4 only(c) 2 and 5 only(d) 1, 2, 3, 4 and 5Sol. 96 (b) 1, 3 and 4 onlySource: PrepMate GeographyPair 1 (Adriatic Sea: Albania) is correctly matched. Source: PrepMate Geography, Chapter 16, Page 178 and 181Pair 2 (Black Sea: Croatia) is not correctly matched. Source: PrepMate Geography, Chapter 16, Page 178Pair 3 (Caspian Sea: Kazakhstan) is correctly matched. Source: PrepMate Geography, Chapter 16, Page 187Pair 4 (Mediterranean Sea: Morocco) is correctly matched. Source: PrepMate Geography, Chapter 16, Page 178Morocco is situated in North Africa.Pair 5 (Red Sea: Syria) is incorrectly matched. Source: PrepMate Geography, Chapter 16, Page 18397. Among the following, which one is the largest exporter of rice in the world in the last five years?(a) China(b) India(c) Myanmar(d) VietnamSol. 97 (b) IndiaSource: PrepMate Geography, Chapter 31, Page 41798. Consider the following pairs:Glacier: River1. Bandarpunch: Yamuna2. Bara Shigri: Chenab3. Milam: Mandakini4. Siachen: Nubra5. Zemu: ManasWhich of the pairs given above are correctly matched?(a) 1, 2 and 4(b) 1, 3 and 4(c) 2 and 5(d) 3 and 5Sol. 98 (a) 1, 2 and 4Topic: GeographyOrigin Glacier: River1. Bandarpunch: Yamuna2. Bara Shigri: Chenab3. Milam: Gori Ganga4. Siachen: Nubra5. Zemu: TeestaPairs 1, 2 and 4 are correctly matched.99. In India, the use of carbofuran, methyl parathion, phorate and triazophos is viewed with apprehension. These chemicals are used as(a) pesticides in agriculture(b) preservatives in processed foods(c) fruit-ripening agents(d) moisturising agents in cosmeticsSol. 99 (a) pesticides in agricultureTopic: Current Affairs100. Consider the following statements:1. Under Ramsar Convention, it is mandatory on the part of the Government of India to protect and conserve all the wetlands in the territory of India.2. The Wetlands (Conservation and Management) Rules, 2010 were framed by the Government of India based on the recommendations of Ramsar Convention.3. The Wetlands (Conservation and Management) Rules, 2010 also encompass the drainage area or catchment regions of the wetlands as determined by the authority.Which of the statements given above is / are correct?(a) 1 and 2 only(b) 2 and 3 only(c) 3 only(d) 1, 2 and 3Sol. 100 (c) 3 onlySource of 1st and 2nd Statement: PrepMate Environment & Biodiversity Book, Chapter 22, Page 319Statement 1 is incorrect.Statement 2 is also incorrect. The Wetlands (Conservation and Management) Rules are framed in 2010 and Ramsar Convention came into existence in the year 1971. Moreover, the agenda of the convention does not mandate nations to frame rules for protection of wetlands.Once we rule out these statements, clearly option (c) is the right answer.For more details, click to download the https://prepmate.in/app/uploads/2019/06/2019-Prelims-GS-Paper-I-most-authentic-answer-key-along-with-detailed-solutions-1.pdf

Why Do Our Customer Upload Us

It's free and easy to use allows me to convert different types of documents of different formats, to pdf converting a pdf document causes the weight to shift besides if I want to share the document they will not be able to make editions which guarantees my authenticity

Justin Miller